1
$\begingroup$

I adopted this question from 2013 Final Entrance Exam on CS.

We have Finite Machine $M$ and Languages $L_1$ to $L_4$ as depicted in following picture:

Image

The question is which of the $A$ to $D$ is correct?

The short answer sheet says $C$ is correct. Anyone could describe how this will be reached?

asked May 15, 2016 at 7:09
$\endgroup$

1 Answer 1

1
$\begingroup$

It's pretty clear from inspecting the DFA that $L_3 = L(M)$: from $q_0$ we do some sequence of 0ドル$-selftransitions (going throught the loop), and then we must see a 1ドル$ to go to $q_1,ドル which is the only terminating state. We could stop then (so then we just have 0ドル^\ast 1$ as the input sequence), or we do excursions to $q_0$ or $q_2,ドル and so we either have 00ドル^\ast 1$ (if we go to $q_0$ and back, maybe taking some loops in-between) or 10ドル$ if we go to $q_2$ and straight back again. Because we can take as many of these "excursions" as we like, and they're disjoint, we get the extra term $(10 + 00^\ast 1)^\ast,ドル giving us exactly $L_3,ドル which is what I would do first (i.e. determine in my own way what the accepted language is, using this kind of reasoning).

Let's look at $L_1$ which is described by 0ドル^\ast 1 (10)^\ast (00^\ast 1 (10)^\ast)^\ast$. If you follow the automaton we get the same 0ドル^\ast 1$ at the start, to get to $q_1$. Then you get optional excursions to $q_2,ドル and then some optional part that can be described as "1 excursion to $q_0$ followed by any number of excursions to $q_2$", and this part can be repeated as many times as we like.

So if we denote an excursion to $q_0$ by $A,ドル and to $q_2$ by $B,ドル then an accepted sequence, after the initial 0ドル^\ast 1$ is of the form $B^\ast (AB^\ast)^\ast$ which is equivalent to $(A + B)^\ast,ドル i.e. any sequence of $A$'s or $B$'s: such a sequence has a certain number $A$'s: if none, we just have $B^\ast,ドル otherwise we start with a number of $B's,ドル and we use one term $A B^\ast$ for every $A$ we encounter. So in fact $L_1 = L_3 = L(M),ドル which rules out option A.

Now both options B and D claim that $L_4$ also describes the DFA. So look at $L_4,ドル which is described by the regular expression: 0ドル^\ast (1 (10)^\ast 0)^\ast 1 (10)^\ast$. So again, any number of $q_0$-loops at the start. Then an expression that goes to $q_1,ドル does any number of excursions to $q_2$ and goes back to $q_0$. We can do the latter any number of times, including never. Then we do a 1ドル,ドル (so we're back at $q_1$) and do any number of excursions to $q_2$ again, and then stop. So all these strings are accepted by the DFA, so certainly $L_4 \subseteq L(M),ドル and this subset is proper, because in $L_4$ we cannot have long strings of 0ドル$'s in later visits to $q_0$: it's not too hard to see that after the initial 0ドル$'s from 0ドル^\ast$ we can never have 3 or more 0ドル's$ in a row. So e.g. the string 100000001ドル \in L(M)$ but not in $L_4$ so the inclusion is proper, $L_4 \subset L(M)$. This shows that option C is correct already. B and D were already out, as $L(M) \neq L_1$ as we saw.

answered May 15, 2016 at 9:13
$\endgroup$
1
  • $\begingroup$ Very useful. I reading ..... $\endgroup$ Commented May 15, 2016 at 9:50

You must log in to answer this question.

Start asking to get answers

Find the answer to your question by asking.

Ask question

Explore related questions

See similar questions with these tags.